Login

Welcome, Guest. Please login or register.

March 29, 2024, 10:16:04 am

Author Topic: VCE Physics Question Thread!  (Read 603395 times)  Share 

0 Members and 4 Guests are viewing this topic.

Robert123

  • Victorian
  • Forum Obsessive
  • ***
  • Posts: 201
  • Respect: +5
  • School: Kyabram P-12 College
Re: VCE Physics Question Thread!
« Reply #345 on: November 09, 2013, 03:35:44 pm »
+2
Henreezy, the reaction force rule is given by
F(a on b)= -F (b on a) that is, newton's 3rd law
From this, we can sort of sub in and solve so we get
F(earth's gravity on object)= -F(object's gravity on earth)
So what actually occur is the gravitation force from the ball actually pulls the earth up but since the earth is so massive, nothing is observed on the earth's behalf.

For young's double slit experiment, does it matter if the light source is coherent or not to achieve interference? I did an exam practice by ITUTE which said that using torch light would not allow interference to occur but in this video by veritasium, he achieved interference with sunlight
http://www.youtube.com/watch?v=Iuv6hY6zsd0

Stevensmay

  • Guest
Re: VCE Physics Question Thread!
« Reply #346 on: November 09, 2013, 03:36:25 pm »
+1
Question 4 b) VCAA 2012 Exam 1:
Is the tension force referred to as the 'gravitational force - upwards' due to the fact that the reaction is due to gravity? I don't know and that confuses me.

Action: Gravity 'pulling' it downwards
Reaction: Gravity 'pushing' it upwards (results in 'tension' from string?)

I'm so confused.

So we agree that the action force is the gravitational attraction exerted on the spheres by the earth.

To find our reactionary force, we simply swap the two objects around.

The reaction force is the gravitational attraction exerted on the earth by the spheres. This is effectively pulling the earth closer to the spheres, thus the direction is up.

Stevensmay

  • Guest
Re: VCE Physics Question Thread!
« Reply #347 on: November 09, 2013, 03:39:50 pm »
+1
For young's double slit experiment, does it matter if the light source is coherent or not to achieve interference? I did an exam practice by ITUTE which said that using torch light would not allow interference to occur but in this video by veritasium, he achieved interference with sunlight
http://www.youtube.com/watch?v=Iuv6hY6zsd0

Someone had the exact same question as you.

http://physics.stackexchange.com/questions/76692/is-coherent-light-required-for-interference-in-youngs-double-slit-experiment

lzxnl

  • Victorian
  • ATAR Notes Legend
  • *******
  • Posts: 3432
  • Respect: +215
Re: VCE Physics Question Thread!
« Reply #348 on: November 09, 2013, 04:56:52 pm »
+1
Henreezy, the reaction force rule is given by
F(a on b)= -F (b on a) that is, newton's 3rd law
From this, we can sort of sub in and solve so we get
F(earth's gravity on object)= -F(object's gravity on earth)
So what actually occur is the gravitation force from the ball actually pulls the earth up but since the earth is so massive, nothing is observed on the earth's behalf.

For young's double slit experiment, does it matter if the light source is coherent or not to achieve interference? I did an exam practice by ITUTE which said that using torch light would not allow interference to occur but in this video by veritasium, he achieved interference with sunlight
http://www.youtube.com/watch?v=Iuv6hY6zsd0

Torch light does allow interference to occur, as you have waves at each slit interfering with each other, but you don't get a particularly convenient interference pattern as you can see in the video.
Question 4 b) VCAA 2012 Exam 1:
Is the tension force referred to as the 'gravitational force - upwards' due to the fact that the reaction is due to gravity? I don't know and that confuses me.

Action: Gravity 'pulling' it downwards
Reaction: Gravity 'pushing' it upwards (results in 'tension' from string?)

I'm so confused.

The tension is merely a force that exists to hold the object there. It doesn't have to exist if the object is falling.
However, as long as there is a gravitational force, there is a reaction to that force, and that force is always the same as the gravitational force in magnitude, but opposite in direction by Newton 3. That's the difference. In this case, as people before me have pointed out, the reaction is the attraction of the Earth by the object.
2012
Mathematical Methods (50) Chinese SL (45~52)

2013
English Language (50) Chemistry (50) Specialist Mathematics (49~54.9) Physics (49) UMEP Physics (96%) ATAR 99.95

2014-2016: University of Melbourne, Bachelor of Science, Diploma in Mathematical Sciences (Applied Maths)

2017-2018: Master of Science (Applied Mathematics)

2019-2024: PhD, MIT (Applied Mathematics)

Accepting students for VCE tutoring in Maths Methods, Specialist Maths and Physics! (and university maths/physics too) PM for more details

Jaswinder

  • Victorian
  • Trendsetter
  • **
  • Posts: 152
  • Respect: 0
  • School Grad Year: 2014
Re: VCE Physics Question Thread!
« Reply #349 on: November 10, 2013, 10:39:11 am »
0
01 - I thought the frequency doesn't change (hence A?) but the answers say C?  :-\

02 -For 14 I thought A, as the left of the battery equipped circuit would become north it would create flux going right to left in the other circuit and to oppose that you would need a current going from X to Y?

Thanks

eddybaha

  • Victorian
  • Trailblazer
  • *
  • Posts: 43
  • Respect: 0
  • School: GWSC
  • School Grad Year: 2013
Re: VCE Physics Question Thread!
« Reply #350 on: November 10, 2013, 10:56:01 am »
0
1. the frequency is changing because the alternator is spinning faster and faster.
2. you are looking for the direction of current through the AMMETER, yes a bit tricky. so when the switch closes, it causes flux to thread the secondary coil to the left. Using lenz's law you need a magnetic field threading the coil to the right to oppose this change in flux. Then using grip rule you get your answer.
Mathematical Methods:44 (2012)
English Language, Specialist Maths, Physics, Chemistry, Extension Maths (monash) (2013)
Hard work beats talent when talent fails to work hard.

This-is-not-me

  • Victorian
  • Forum Regular
  • **
  • Posts: 54
  • Respect: 0
Re: VCE Physics Question Thread!
« Reply #351 on: November 10, 2013, 01:03:34 pm »
0
Can someone help me with the concept in part of b of this question?
2013:
English | Methods | Physics | IT:Applications

eddybaha

  • Victorian
  • Trailblazer
  • *
  • Posts: 43
  • Respect: 0
  • School: GWSC
  • School Grad Year: 2013
Re: VCE Physics Question Thread!
« Reply #352 on: November 10, 2013, 01:41:33 pm »
0
because an electron has a debroglie wavelength it forms standing waves in orbit. the circumference of the orbit can only be an integer number of wavelengths and thus only these energy levels are stable.
Mathematical Methods:44 (2012)
English Language, Specialist Maths, Physics, Chemistry, Extension Maths (monash) (2013)
Hard work beats talent when talent fails to work hard.

Alwin

  • Victorian
  • Forum Leader
  • ****
  • Posts: 838
  • Respect: +241
Re: VCE Physics Question Thread!
« Reply #353 on: November 10, 2013, 01:44:41 pm »
0
Can someone help me with the concept in part of b of this question?

One (vce level reply) would be to say that electrons need to form stationary waves to be stable so the orbit circumference has to be a whole number multiple of the wavelength of the electron's de broglie wavelength.
If an electron orbited at radius such that the circumference was not a whole number multiple, destructive interference would occur and the electron would 'drop' to a lower stable orbit.
Thus, only certain energy levels, orbits, are possible for the electrons of the hydrogen atom... or something along those lines

inb4 nliu comments how they're not levels so much, but bands but that's beyond vce
2012:  Methods [48] Physics [49]
2013:  English [40] (oops) Chemistry [46] Spesh [42] Indo SL [34] Uni Maths: Melb UMEP [4.5] Monash MUEP [just for a bit of fun]
2014:  BAeroEng/BComm

A pessimist says a glass is half empty, an optimist says a glass is half full.
An engineer says the glass has a safety factor of 2.0

lzxnl

  • Victorian
  • ATAR Notes Legend
  • *******
  • Posts: 3432
  • Respect: +215
Re: VCE Physics Question Thread!
« Reply #354 on: November 10, 2013, 02:19:11 pm »
+1
One (vce level reply) would be to say that electrons need to form stationary waves to be stable so the orbit circumference has to be a whole number multiple of the wavelength of the electron's de broglie wavelength.
If an electron orbited at radius such that the circumference was not a whole number multiple, destructive interference would occur and the electron would 'drop' to a lower stable orbit.
Thus, only certain energy levels, orbits, are possible for the electrons of the hydrogen atom... or something along those lines

inb4 nliu comments how they're not levels so much, but bands but that's beyond vce

Actually, I'm happy with calling them energy levels; I'm not happy with the standing wave explanation in general :P
It only works for any atom with one electron. AKA H, He+, Li 2+ etc

Alwin, I think you might have say that as only specific wavelengths are permitted, only specific energies are allowed. This is VCE. We can't make any jumps in our thought processes apparently.
2012
Mathematical Methods (50) Chinese SL (45~52)

2013
English Language (50) Chemistry (50) Specialist Mathematics (49~54.9) Physics (49) UMEP Physics (96%) ATAR 99.95

2014-2016: University of Melbourne, Bachelor of Science, Diploma in Mathematical Sciences (Applied Maths)

2017-2018: Master of Science (Applied Mathematics)

2019-2024: PhD, MIT (Applied Mathematics)

Accepting students for VCE tutoring in Maths Methods, Specialist Maths and Physics! (and university maths/physics too) PM for more details

Robert123

  • Victorian
  • Forum Obsessive
  • ***
  • Posts: 201
  • Respect: +5
  • School: Kyabram P-12 College
Re: VCE Physics Question Thread!
« Reply #355 on: November 11, 2013, 09:56:30 pm »
0
Is diffraction stronger when wavelength equals slit width or when wavelength is greater than slit width?

lzxnl

  • Victorian
  • ATAR Notes Legend
  • *******
  • Posts: 3432
  • Respect: +215
Re: VCE Physics Question Thread!
« Reply #356 on: November 11, 2013, 10:00:04 pm »
0
Technically speaking, it's stronger when the wavelength is bigger than the slit width as the intensity of the diffracted wave then decreases more slowly as you get further from the intensity maximum at the middle, but for VCE purposes I'm not sure how relevant that is.
2012
Mathematical Methods (50) Chinese SL (45~52)

2013
English Language (50) Chemistry (50) Specialist Mathematics (49~54.9) Physics (49) UMEP Physics (96%) ATAR 99.95

2014-2016: University of Melbourne, Bachelor of Science, Diploma in Mathematical Sciences (Applied Maths)

2017-2018: Master of Science (Applied Mathematics)

2019-2024: PhD, MIT (Applied Mathematics)

Accepting students for VCE tutoring in Maths Methods, Specialist Maths and Physics! (and university maths/physics too) PM for more details

BasicAcid

  • Victorian
  • Forum Obsessive
  • ***
  • Posts: 207
  • Respect: +135
Re: VCE Physics Question Thread!
« Reply #357 on: November 11, 2013, 10:55:52 pm »
0
Is diffraction stronger when wavelength equals slit width or when wavelength is greater than slit width?

For VCE purposes, they are the same strength.

Infinity Plus

  • Victorian
  • Adventurer
  • *
  • Posts: 15
  • Respect: 0
Re: VCE Physics Question Thread!
« Reply #358 on: November 12, 2013, 04:22:56 pm »
0
Hey guys. With regards to diffraction I have read many sources that say different things so I would like to seek the opinions of my fellow trustworthy ANers. How large does the ratio of lambda/width have to be for there to be significant diffraction? MOst sources say ,1 others say a bit less than 1 is still considered a lot, and few say more than one.
2012: Biology [43]
2013: Specialist | Methods | English | Physics | Chemistry

eddybaha

  • Victorian
  • Trailblazer
  • *
  • Posts: 43
  • Respect: 0
  • School: GWSC
  • School Grad Year: 2013
Re: VCE Physics Question Thread!
« Reply #359 on: November 12, 2013, 04:31:31 pm »
+1
definitely noticeable diffraction at 1 but vcaa will either give you something obviously small like 0.000001 if theres no diffraction
Mathematical Methods:44 (2012)
English Language, Specialist Maths, Physics, Chemistry, Extension Maths (monash) (2013)
Hard work beats talent when talent fails to work hard.